MODULE 9 CAPITAL BUDGETING THEORIES: Basic Concepts

38
Capital Budgeting MODULE 9 CAPITAL BUDGETING THEORIES: Basic Concepts Decision Making Process 2. The first step in the decision-making process is to A. determine and evaluate possible courses of action. B. identify the problem and assign responsibility. C. make a decision. D. review results of the decision. Strategic planning 39.Strategic planning is the process of deciding on an organization’ A. minor programs and the approximate resources to be devoted to them B. major programs and the approximate resources to be devoted to them C. minor programs prior to consideration of resources that might be needed D. major programs prior to consideration of resources that might be needed Capital budgeting defined 1. The long-term planning process for making and financing investments that affect a company’s financial results over a number of years is referred to as A. capital budgeting C. master budgeting B. strategic planning D. long-range planning 3. Capital budgeting is the process A. used in sell or process further decisions. B. of determining how much capital stock to issue C. of making capital expenditure decisions D. of eliminating unprofitable product line 5. A capital investment decision is essentially a decision to: A. exchange current assets for current liabilities. B. exchange current cash outflows for the promise of receiving future cash inflows. C. exchange current cash flow from operating activities for future cash inflows from investing activities. D. exchange current cash inflows for future cash outflows. Risk & return 6. The higher the risk element in a project, the A. more attractive the investment is. B. higher the net present value is. C. higher the cost of capital is. D. higher the discount rate is. 9. Cost of capital is the A. amount the company must pay for its plant assets. B. dividends a company must pay on its equity securities. C. cost the company must incur to obtain its capital resources. D. cost the company is charged by investment bankers who handle the issuance of equity or long-term debt securities. 14.How should the following projects be listed in order of increasing risk? A. New venture, replacement, expansion. B. Replacement, new venture, expansion. C. Replacement, expansion, new venture. D. Expansion, replacement, new venture. 41.Problems associated with justifying investments in high-tech projects often include discount rates that are too A. low and time horizons that are too long B. high and time horizons that are too long 227

Transcript of MODULE 9 CAPITAL BUDGETING THEORIES: Basic Concepts

Page 1: MODULE 9 CAPITAL BUDGETING THEORIES: Basic Concepts

Capital Budgeting

MODULE 9

CAPITAL BUDGETING

THEORIES:Basic ConceptsDecision Making Process2. The first step in the decision-making process is to

A. determine and evaluate possible courses of action.B. identify the problem and assign responsibility.C. make a decision.D. review results of the decision.

Strategic planning39.Strategic planning is the process of deciding on an organization’

A. minor programs and the approximate resources to be devoted to themB. major programs and the approximate resources to be devoted to themC. minor programs prior to consideration of resources that might be neededD. major programs prior to consideration of resources that might be needed

Capital budgeting defined1. The long-term planning process for making and financing investments that affect a

company’s financial results over a number of years is referred to asA. capital budgeting C. master budgetingB. strategic planning D. long-range planning

3. Capital budgeting is the processA. used in sell or process further decisions.B. of determining how much capital stock to issueC. of making capital expenditure decisionsD. of eliminating unprofitable product line

5. A capital investment decision is essentially a decision to:A. exchange current assets for current liabilities.B. exchange current cash outflows for the promise of receiving future cash inflows.C. exchange current cash flow from operating activities for future cash inflows from

investing activities.D. exchange current cash inflows for future cash outflows.

Risk & return6. The higher the risk element in a project, the

A. more attractive the investment is.B. higher the net present value is.C. higher the cost of capital is.D. higher the discount rate is.

9. Cost of capital is theA. amount the company must pay for its plant assets.B. dividends a company must pay on its equity securities.C. cost the company must incur to obtain its capital resources.D. cost the company is charged by investment bankers who handle the issuance of

equity or long-term debt securities.

14.How should the following projects be listed in order of increasing risk?A. New venture, replacement, expansion.B. Replacement, new venture, expansion.C. Replacement, expansion, new venture.D. Expansion, replacement, new venture.

41.Problems associated with justifying investments in high-tech projects often includediscount rates that are tooA. low and time horizons that are too longB. high and time horizons that are too long

227

Page 2: MODULE 9 CAPITAL BUDGETING THEORIES: Basic Concepts

Capital Budgeting

C. high and time horizons that are too shortD. low and time horizons that are too short

60.In evaluating high-tech projects,A. only tangible benefits should be considered.B. only intangible benefits should be considered.C. both tangible and intangible benefits should be considered.D. neither tangible nor intangible benefits should be considered.

Types of capital projects4. A project that when accepted or rejected will not affect the cash flows of another

project.A. Independent projects C. Mutually exclusive projectsB. Dependent projects D. Both b and c

Capital budgeting process7. The normal methods of analyzing investments

A. cannot be used by not-for-profit entities.B. do not apply if the project will not produce revenues.C. cannot be used if the company plans to finance the project with funds already

available internally.D. require forecasts of cash flows expected from the project.

InvestmentsSale of old asset38.When disposing of an old asset and replacing it with a new one, tax effect on

A. gain on sale of the old asset reduces the basis of the new assetB. gain on sale of the old asset increases the basis of the new assetC. loss on sale of the old asset reduces the basis of the new assetD. b and c

Working capital18.A major difference between an investment in working capital and one in depreciable

assets is thatA. an investment in working capital is never returned, while most depreciable assets

have some residual value.B. an investment in working capital is returned in full at the end of a project’s life,

while an investment in depreciable assets has no residual value.C. an investment in working capital is not tax-deductible when made, nor taxable

when returned, while an investment in depreciable assets does allow taxdeductions.

D. because an investment in working capital is usually returned in full at the end of theproject’s life, it is ignored in computing the amount of the investment required forthe project.

30.The proper treatment of an investment in receivables and inventory is toA. ignore itB. add it to the required investment in fixed assetsC. add it to the required investment in fixed assets and subtract it from the annual

cash flowsD. add it to the investment in fixed assets and add the present value of the recovery to

the present value of the annual cash flows

31.In connection with a capital budgeting project, an investment in working capital isnormally recoveredA. at the end of the project’s lifeB. in the first year of the project’s lifeC. evenly through the project’s lifeD. when the company goes out of businessA

32.XYZ Co. is adopting just-in-time principles. When evaluating an investment project thatwould reduce inventory, how should XYZ treat the reduction?A. Ignore it.

228

Page 3: MODULE 9 CAPITAL BUDGETING THEORIES: Basic Concepts

Capital Budgeting

B. Decrease the cost of the investment and decrease cash flows at the end of theproject’s life.

C. Decrease the cost of the investment.D. Decrease the cost of the investment and increase the cash flow at the end of the

project’s life.

Relevant cash flows72.Which of the following represents the biggest challenge in the decision to purchase

new equipment?A. Estimating employee training for the new project.B. Estimating cash flows for the future.C. Estimating transportation costs of the new equipment.D. Estimating maintenance costs for the new equipment.

51.When a firm has the opportunity to add a project that will utilize factory capacity that iscurrently not being used, which costs should be used to determine if the added projectshould be undertaken?A. Opportunity costs C. Net present costsB. Historical costs D. Incremental costs

11.The only future costs that are relevant to deciding whether to accept an investment arethose that willA. be different if the project is accepted rather than rejected.B. be saved if the project is accepted rather than rejected.C. be deductible for tax purposes.D. affect net income in the period that they are incurred.

Cash inflow66.Which of the following is not a typical cash inflow in capital investment decisions?

A. Incremental revenues C. Salvage valueB. Cost reductions D. Additional working capital

Out-of-pocket costs45.Which of the following is a cost that requires a future outlay of cash that is which

relevant for future decision-making?A. Opportunity cost C. Sunk costsB. Out-of-pocket cost D. Relevant benefits

Depreciation & Tax 22.If there were no income taxes,

A. depreciation would be ignored in capital budgeting.B. the NPV method would not work.C. income would be discounted instead of cash flow.D. all potential investments would be desirable.

21.Relevant cash flows for net present value (NPV) models include all of the followingexceptA. outflows to purchase new equipmentB. depreciation expense on the newly acquired piece of equipmentC. reductions in operating cash flows as a result of using the new equipment.D. cash outflows related to purchasing additional inventories for another retail store.

55.When evaluating depreciation methods, managers who are concerned about capitalinvestment decisions will:A. choose straight line depreciation so there is minimum impact on the decision.B. use units of production so more depreciation expense will be allocated to the later

years.C. use accelerated methods to have as much depreciation in the early years of an

asset’s life.D. choice of depreciation method has no impact on the capital investment decision.

70.The tax consequences should be considered under which circumstances when makingcapital investment decisions?

229

Page 4: MODULE 9 CAPITAL BUDGETING THEORIES: Basic Concepts

Capital Budgeting

A. Positive net income C. DepreciationB. Disposal of an asset D. All of the above

Irrelevant cash flowsLoan financing43.In addition to incremental revenues, cash inflows from capital investments can be

generated from all of the following sources except:A. debt financingB. cost savingsC. salvage valueD. reduction in the amount of working capital

10.If Helena Company expects to get a one-year bank loan to help cover the initialfinancing of one of its capital projects, the analysis of the project shouldA. offset the loan against any investment in inventory or receivables required by the

project.B. show the loan as an increase in the investment.C. show the loan as a cash outflow in the second year of the project’s life.D. ignore the loan

Sunk cost29.In deciding whether to replace a machine, which of the following is NOT a sunk cost?

A. The expected resale price of the existing machine.B. The book value of the existing machine.C. The original cost of the existing machine.D. The depreciated cost of the existing machine.

Accounting rate of return54.The primary advantages of the average rate of return method are its ease of

computation and the fact that:A. It is especially useful to managers whose primary concern is liquidityB. There is less possibility of loss from changes in economic conditions and

obsolescence when the commitment is short-termC. It emphasizes the amount of income earned over the life of the proposalD. Rankings of proposals are necessary

Nondiscounted cash flow methodPayback method36.There are several capital budgeting decision models that do not use discounted cash

flows. What is the name of the simple technique that calculates the total time it willtake to recover, using cash inflows from operations, the amount of cash invested in aproject?A. Recovery period C. External rate of returnB. Payback model D. Accounting rate of return

34.The technique most concerned with liquidity isA. Payback method.B. Net present value technique.C. Internal rate of return.D. book rate of return.

73.Which of the following is a potential use of the payback method?A. Help managers control the risks of estimating cash flowsB. Help minimize the impact of the investment on liquidityC. Help control the risk of obsolescenceD. All of the answers are correct

47.The cash payback technique:A. should be used as a final screening tool.B. can be the only basis for the capital budgeting decision.C. is relatively easy to compute and understand.D. considers the expected profitability of a project.

230

Page 5: MODULE 9 CAPITAL BUDGETING THEORIES: Basic Concepts

Capital Budgeting

33.Which of the following is NOT a defect of the payback method?A. It ignores cash flows because it uses net income.B. It ignores profitability.C. It ignores the present values of cash flows.D. It ignores the pattern of cash flows beyond the payback period.

48.The payback method, as a capital budgeting technique, assumes that all intermediatecash inflows are reinvested to yield a return equal to:A. Zero C. The Discount RateB. The Time-Adjusted-Rate-of-Return D. The Cost-of-Capital

52.Which of the following capital budgeting methods is the least theoretically correct?A. payback method C. internal rate of returnB. net present value D. none of the above

Discounted cash flow method49.Which of the following methods of evaluating capital investment projects incorporates

the time value of money?A. Payback period, accounting rate of return, and internal rate of returnB. Accounting rate of return, net present value, and internal rate of returnC. Payback period and accounting rate of returnD. Net present value and internal rate of return

Net present value69.Discounted cash flow analysis is used in which of the following techniques?

A. Net present value C. Cost of capitalB. Payback period D. All of the above

8. The primary capital budgeting method that uses discounted cash flow techniques is theA. net present value method.B. cash payback technique.C. annual rate of return method.D. profitability index method.

20.The net present value (NPV) model can be used to evaluate and rank two or moreproposed projects. The approach that computes the total impact on cash flows for eachoption and then converts these total cash flows to their present values is called theA. differential approach C. contribution approachB. incremental approach. D. total project approach.

40.The discount rate commonly used in present value calculations is theA. treasury bill rateB. weighted average return on assets adjusted for riskC. risk free rate plus inflation rateD. shareholders’ expected return on equity

44.Which is true of the net present value method of determining the acceptability of aninvestment?A. The initial cost of the investment is subtracted from the present value of net cash

flowsB. The net cash flows are not adjusted to present valueC. A negative net present value indicates the investment should be undertakenD. The net present value method requires no subjective judgments

Profitability index35.The profitability index

A. does not take into account the discounted cash flows.B. Is calculated by dividing total cash flows by the initial investment.C. allows comparison of the relative desirability of projects that require differing initial

investments.D. will never be greater than 1.0.

Internal rate of return

231

Page 6: MODULE 9 CAPITAL BUDGETING THEORIES: Basic Concepts

Capital Budgeting

56.According to the reinvestment rate assumption, which method of capital budgetingassumes cash flows are reinvested at the project’s rate of return?A. payback period C. internal rate of returnB. net present value D. none of the above

62.The rate of interest that produces a zero net present value when a project’s discountedcash operating advantage is netted against its discounted net investment is the:A. Cost of capital C. Cutoff rateB. Discount rate D. Internal rate of return

57.A weakness of the internal rate of return method for screening investment projects isthat it:A. Does not consider the time value of moneyB. Implicitly assumes that the company is able to reinvest cash flows from the

project at the company’s discount rateC. Implicitly assumes that the company is able to reinvest cash flows from the project

at the internal rate of returnD. Fails to consider the timing of cash flows

Comprehensive50.Which of the following methods of evaluating capital investment projects do not use a

percentage as a measurement unit?A. Payback period and net present valueB. Accounting rate of return and payback periodC. Net present value and internal rate of returnD. Internal rate of return and payback period

Relationships among NPV, PI & IRR24.If a company’s required rate of return is 12 percent and in using the profitability index

method, a project’s index is greater than 1.0, this indicates that the project’s rate ofreturn isA. equal to 12 percent. C. less than 12 percent.B. greater than 12 percent. D. dependent on the size of the investment.

25.If the present value of the future cash flows for an investment equals the requiredinvestment, the IRR isA. equal to the cutoff rate.B. equal to the cost of borrowed capital.C. equal to zero.D. lower than the company’s cutoff rate return.

27.The relationship between payback period and IRR is thatA. a payback period of less than one-half the life of a project will yield an IRR lower

than the target rate.B. the payback period is the present value factor for the IRR.C. a project whose payback period does not meet the company’s cutoff rate for

payback will not meet the company’s criterion for IRR.D. none of the above.

67.When comparing NPV and IRR, which is not true?A. With NPV, the discount rate can be adjusted to take into account increased risk and

the uncertainty of cash flowsB. With IRR, cash flows can be adjusted to account for riskC. NPV can be used to compare investments of various size or magnitudeD. Both NPV and IRR can be used for screening decisions

Sensitivity analysis13.In capital budgeting, sensitivity analysis is used

A. to determine whether an investment is profitable.B. to see how a decision would be affected by changes in variables.C. to test the relationship of the IRR and NPV.D. to evaluate mutually exclusive investments.

232

Page 7: MODULE 9 CAPITAL BUDGETING THEORIES: Basic Concepts

Capital Budgeting

15.An approach that uses a number of outcome estimates to get a sense of the variabilityamong potential returns isA. the discounted cash flow technique.B. the net present value method.C. risk analysis.D. sensitivity analysis.

42.Sensitivity analysis is the study of how the outcome of a decision making processA. changes as one or more of the assumptions changeB. remains the same even though one or more of the assumptions changeC. changes even though one or more of the assumptions do not changeD. does not change as the assumptions do not change either

64.Sensitivity analysis is:A. An appropriate response to uncertainty in cash flow projectionsB. Useful in measuring the variance of the Fisher rateC. Typically conducted in the post investment auditD. Useful to compare projects requiring vastly different levels of initial investment

IRR = 058.if the internal rate of return on an investment is zero:

A. its NPV is positive.B. its annual cash flows equal its required investment.C. it is generally a wise investment.D. its cash flows decrease over its life.

Change in NPV59.Which of the following would decrease the net present value of a project?

A. A decrease in the income tax rateB. A decrease in the initial investmentC. An increase in the useful life of the projectD. An increase in the discount rate

Effect of change in cost of capital26.All other things being equal, as cost of capital increases

A. more capital projects will probably be acceptable.B. fewer capital projects will probably be acceptable.C. the number of capital projects that are acceptable will change, but the direction of

the change is not determinable just by knowing the direction of the change in costof capital.

D. the company will probably want to borrow money rather than issue stock.

Effect of change in residual value23.Assuming that a project has already been evaluated using the following techniques, the

evaluation under which technique is least likely to be affected by an increase in theestimated residual value of the project?A. Payback Period. C. Net Present Value.B. Internal Rate of Return. D. Profitability Index.

Decision rules – independent projects68.What type of decision involves deciding if an investment meets a predetermined

standard?A. Investment decisions C. Management decisionsB. Screening decisions D. Preference decisions

Payback period46.If a payback period for a project is greater than its expected useful life, the

A. project will always be profitable.B. entire initial investment will not be recovered.C. project would only be acceptable if the company’s cost of capital was low.D. project’s return will always exceed the company’s cost of capital.

Net present value

233

Page 8: MODULE 9 CAPITAL BUDGETING THEORIES: Basic Concepts

Capital Budgeting

61.An analysis of a proposal by the net present value method indicated that the presentvalue of future cash inflows exceeded the amount to be invested. Which of thefollowing statements best describes the results of this analysis?A. The proposal is desirable and the rate of return expected from the proposal exceeds

the minimum rate used for the analysisB. The proposal is desirable and the rate of return expected from the proposal is less

than the minimum rate used for the analysisC. The proposal is undesirable and the rate of return expected from the proposal is less

than the minimum rate used for the analysisD. The proposal is undesirable and the rate of return expected from the proposal

exceeds the minimum rate used for the analysis

63.NPV indicates a project is deemed desirable (acceptable) when the NPV isA. greater than or equal to zeroB. less than zeroC. greater than or equal to the risk-adjusted cost of capitalD. less than or equal to the risk-adjusted cost of capital

Internal rate of return12.If Arbitrary Company wants to use IRR to evaluate long-term decisions and to establish

a cutoff rate of return, it must be sure that the cutoff rate isA. at least equal to its cost of capital.B. at least equal to the rate used by similar companies.C. greater than the IRR on projects accepted in the past.D. greater than the current book rate of return.

NPV & IRR19.The NPV and IRR methods give

A. the same decision (accept or reject) for any single investment.B. the same choice from among mutually exclusive investments.C. different rankings of projects with unequal lives.D. the same rankings of projects with different required investments.

Decision rule – mutually exclusive projects71.Mutually exclusive projects are those that:

A. if accepted, preclude the acceptance of competing projects.B. if accepted, can have a negative effect on the company’s profit.C. if accepted, can also lead to the acceptance of a competing project.D. require all managers to consider.

28.In choosing from among mutually exclusive investments the manager should normallyselect the one with the highestA. Net present value. C. Profitability index.B. Internal rate return. D. Book rate of return.

53.Why do the NPV method and the IRR method sometimes produce different rankings ofmutually exclusive investment projects?A. The NPV method does not assume reinvestment of cash flows while the IRR method

assumes the cash flows will be reinvested at the internal rate of return.B. The NPV method assumes a reinvestment rate equal to the discount rate while the

IRR method assumes a reinvestment rate equal to the internal rate of return.C. The IRR method does not assume reinvestment of the cash flows while the NPV

assumes the reinvestment rate is equal to the discount rate.D. The NPV method assumes a reinvestment rate equal to the bank loan interest rate

while the IRR method assumes a reinvestment rate equal to the discount rate.

Post-audit16.Post-audit of capital projects

A. is usually conclusive.B. is done using different evaluation techniques than were used in making the original

capital budgeting decision.C. provides a formal mechanism by which the company can determine whether

existing projects should be supported or terminated.

234

Page 9: MODULE 9 CAPITAL BUDGETING THEORIES: Basic Concepts

Capital Budgeting

D. all of the above.

17.A thorough evaluation of how well a project’s actual performance matches theprojections made when the project was proposed is called aA. pre-audit. C. sensitivity analysis.B. post-audit. D. risk analysis.

37.A follow-up evaluation of a capital project is performed to see that investmentexpenditures are proceeding on time and on budget, to compare actual cash flows withthose originally predicted, and to evaluate continuation of the project. This follow-up iscalled aA. postaudit. C. management auditB. performance evaluation D. project review

65.Companies use post audits to:A. chastise managers whose project does not exceed projections.B. prove to managers that they should have accepted projects they previously

rejected.C. have the managers revise poorly performing projects so the projects will have larger

return in the future.D. provide feedback that enables managers to improve the accuracy of the projections

of future cash flows, thereby maximizing the quality of the firm’s capitalinvestments.

PROBLEMS:Net Investmenti. Bruell Company is considering to replace its old equipment with a new one. The old

equipment had a net book value of P100,000, 4 remaining useful life with P25,000depreciation each year. The old equipment can be sold at P80,000. The newequipment costs P160,000, have a 4-year life. Cash savings on operating expensesbefore 40% taxes amount to P50,000 per year. What is the amount of investment inthe new equipment?A. P160,000 C. P 80,000B. P 72,000 D. P 68,000

Operating Cash FlowCash Flow Before taxii. Taal Company is considering the purchase of a machine that promises to reduce

operating costs by equal amounts every year of its 6-year useful life. The machine willcost P840,000 and has no salvage value. The machine has a 20% internal rate ofreturn. Taal Company is subject to 40% income tax rate. The present value of 1 for 6periods at 20% is 3.326, and at the end of 6 periods is 0.3349. The approximate annual cash savings before tax is closest to:A. P252,555 C. P187,592B. P112,555 D. P327,592

Increase in Annual Income Taxiii. Mayon Company is considering replacing its old machine with a new and more efficient

one. The old machine has book value of P100,000, a remaining useful life of 4 years,and annual straight-line depreciation of P25,000. The existing machine has a currentmarket value of P80,000. The replacement machine would cost P160,000, have a 4-year life, and will save P50,000 per year in cash operating costs. If the replacementmachine would be depreciated using the straight-line method and the tax rate is 40%,what should be the increase in annual income taxes?A. P14,000 C. P40,000B. P28,000 D. P 4,000

Depreciation & Taxesiv. Prime Consulting, Inc. operates consulting offices in Manila, Olongapo, and Cebu. The

firm is presently considering an investment in a new mainframe computer andcommunication software. The computer would cost P6 million and have an expectedlife of 8 years. For tax purposes, the computer can be depreciated using either

235

Page 10: MODULE 9 CAPITAL BUDGETING THEORIES: Basic Concepts

Capital Budgeting

straight-line method or Sum-of-the-Years’-Digits (SYD) method over five years. Nosalvage value is recognized in computing depreciation expense and no salvage value isexpected at the end of the life of the equipment. The company’s cost of capital is 10percent and its tax rate is 40 percent.The present value of annuity of 1 for 5 periods is 3.791 and for 8 periods is 5.335. Thepresent values of 1 end of each period are:

1 0.9091 5 0.62092 0.8264 6 0.56453 0.6513 7 0.51324 0.6830 8 0.4665

The present value of the net advantage of using SYD method of depreciation with afive-year life instead of straight-line method of depreciating the equipment is:A. P 86,224 C. P215,560B. P115,168 D. P287,893

v. For P450,000, Maleen Corporation purchased a new machine with an estimated usefullife of five years with no salvage value. The machine is expected to produce cash flowfrom operations, net of 40 percent income taxes, as follows:

First year P160,000Second year 140,000Third year 180,000Fourth year 120,000Fifth year 100,000

Maleen will use the sum-of-the-years-digits’ method to depreciate the new machine asfollows:

First year P150,000Second year 120,000Third year 90,000Fourth year 60,000Fifth year 30,000

The present value of 1 for 5 periods at 12 percent is 3.60478. The present values of 1at 12 percent at end of each period are:

End of: Period 1 0.89280Period 2 0.79719Period 3 0.71178Period 4 0.63552Period 5 0.56743

Had Maleen used straight-line method of depreciation instead of declining method,what is the difference in net present value provided by the machine at a discount rateof 12 percent?A. Increase of P 9,750 C. Decrease of P24,376B. Decrease of P 9,750 D. Increase of P24,376

Accounting rate of returnBased on initial investmentvi. A piece of labor saving equipment that Marubeni Electronics Company could use to

reduce costs in one of its plants in Angeles City has just come onto the market.Relevant data relating to the equipment follow:

Purchase cost of the equipment P432,000Annual cost savings that will be provided by the equipment90,000Life of the equipment 12 years

What is the simple rate of return to be provided by the equipment?A. Between 15% and 18%. C. 20.83%.B. 25.00%. D. 12.50%.

Based on average investmentvii. The BIBO Company has made an investment in video and recording equipment that

costs P106,700. The equipment is expected to generate cash inflows of P20,000 peryear. How many years will the equipment have to be used to provide the companywith a 10 percent average accounting rate of return on its investment?A. 7.28 years C. 9.05 yearsB. 5.55 years D. 4.75 years

236

Page 11: MODULE 9 CAPITAL BUDGETING THEORIES: Basic Concepts

Capital Budgeting

viii. Show Company is negotiating to purchase an equipment that would cost P200,000,with the expectation that P40,000 per year could be saved in after-tax cash operatingcosts if the equipment were acquired. The equipment’s estimated useful life is 10years, with no salvage value, and would be depreciated by the straight-line method.Show Company’s minimum desired rate of return is 12 percent. The present value ofan annuity of 1 at 12 percent for 10 periods is 5.65. The present value of 1 due in 10periods, at 12 percent, is 0.322.The average accrual accounting rate of return (ARR) during the first year of asset’s useis:A. 20.0 percent C. 10.0 percentB. 10.5 percent D. 40.0 percent

ix. An asset was purchased for P66,000. The asset is expected to last for 6 years and willhave a salvage value of P16,000. The company expects the income before tax to beP7,200 and the tax rate applicable to the company is 30%. What is the average returnon investment (accounting rate of return)?A. 17.6% C. 10.9%B. 7.6% D. 12.3%

Net Investmentx. The Makabayan Company is planning to purchase a new machine which it will

depreciate, for book purposes, on a straight-line basis over a ten-year period with nosalvage value and a full year’s depreciation taken in the year of acquisition. The newmachine is expected to produce cash flows from operations, net of income taxes, ofP66,000 a year in each of the next ten years. The accounting (book value) rate ofreturn on the initial investment is expected to be 12 percent. How much will the newmachine cost?A. P300,000 C. P550,000B. P660,000 D. P792,000

xi. The Fields Company is planning to purchase a new machine which it will depreciate, forbook purposes, on a straight-line basis over a ten-year period with no salvage valueand a full year’s depreciation taken in the year of acquisition. The new machine isexpected to produce cash flow from operations, net of income taxes, of P66,000 a yearin each of the next ten years. The accounting (book value) rate of return on the initialinvestment is expected to be 12%. How much will the new machine cost?A. P300,000 C. P660,000B. P550,000 D. P792,000

CFATxii. The Hills Company, a calendar company, purchased a new machine for P280,000 on

January 1. Depreciation for tax purposes will be P35,000 annually for eight years. Theaccounting (book value) rate of return (ARR) is expected to be 15% on the initialincrease in required investment. On the assumption of a uniform cash inflow, thisinvestment is expected to provide annual cash flow from operations, net of incometaxes, ofA. P35,000 C. P42,000B. P40,250 D. P77,000

Payback Periodxiii. If an asset costs P35,000 and is expected to have a P5,000 salvage value at the end of

its ten-year life, and generates annual net cash inflows of P5,000 each year, the cashpayback period isA. 8 years C. 6 yearsB. 7 years D. 5 years

xiv. Consider a project that requires cash outflow of P50,000 with a life of eight years and asalvage value of P5,000. Annual before-tax cash inflow amounts to P10,000 assuming atax rate of 30% and a required rate of return of 8%. Salvage value is ignored incomputing depreciation. The project has a payback period ofA. 5.0 years C. 6.0 yearsB. 5.6 years D. 6.6 years

237

Page 12: MODULE 9 CAPITAL BUDGETING THEORIES: Basic Concepts

Capital Budgeting

xv. The following incomplete information is provided for an investment decision.

Year Cash FlowDiscount

Factor(10%)

Discounted CashFlows

CumulativeCash Flows

0 P(450,000) 1.000 P(450,000) P(450,000)1 280,000 .909 254,5202 210,000 .8263 140,000 .751

Using break-even time (BET) analysis, when will the investment be recovered?A. In 2.73 years C. At the end of year 2B. Longer than three years D. In 2.21 years

xvi. Orlando Corporation is considering an investment in a new cheese-cutting machine toreplace its existing cheese cutter. Information on the existing machine and thereplacement machine follow:

Cost of the new machine P400,000Net annual savings in operating costs 90,000Salvage value now of the old machine 60,000Salvage value of the old machine in 8 years 0Salvage value of the new machine in 8 years 50,000Estimated life of the new machine 8 years

What is the expected payback period for the new machine?A. 4.44 years C. 2.67 yearsB. 8.50 years D. 3.78 years

xvii. For P4,500,000, Siniloan Corporation purchased a new machine with an estimateduseful life of five years with no salvage value at its retirement. The machine isexpected to produce cash flow from operations, net of income taxes, as follows:

First year P 900,000Second year 1,200,000Third year 1,500,000Fourth year 900,000Fifth year 800,000

Siniloan will use the sum-of-the-years-digits’ method to depreciate the new machine asfollows:

First year P1,500,000Second year 1,200,000Third year 900,000Fourth year 600,000Fifth year 300,000

What is the payback period for the machine?A. 3 years C. 5 yearsB. 4 years D. 2 years

xviii. Paz Insurance Company’s management is considering an advertising program thatwould require an initial expenditure of P165,500 and bring in additional sales over thenext five years. The cost of advertising is immediately recognized as expense. Theprojected additional sales revenue in Year 1 is P75,000, with associated expenses ofP25,000. The additional sales revenue and expenses from the advertising program areprojected to increase by 10 percent each year. Paz Insurance Company’s tax rate is 40percent.The payback period for the advertising program is A. 4.6 years C. 3.0 yearsB. 1.9 years D. 2.5 years

xix. The Leisure Company is considering the purchase of electronic pinball machines toplace in amusement houses. The machines would cost a total of P300,000, have aneight-year useful life, and have a total salvage value of P20,000. Based on experiencewith other equipment, the company estimates that annual revenues and expensesassociated with the machines would be as follows:

Revenues form use P200,000Less operating expenses

238

Page 13: MODULE 9 CAPITAL BUDGETING THEORIES: Basic Concepts

Capital Budgeting

Commissions to amusement houses P100,000 Insurance 7,000 Depreciation 35,000 Maintenance 18,000 160,000Net income P 40,000

Ignoring the effect of income taxes, the payback period for the pinball machines wouldbeA. 3.73 years C. 4.0 years B. 3.23 years D. 7.5 years

Net Present Valuexx. It is the start of the year and Agudelo Company plans to replace its old grinding

equipment. The following information are made available by the management:Old New

Equipment cost P70,000 P120,000Current salvage value 14,000 -Salvage value, end ofuseful life

5,000 16,000

Annual operating costs 44,000 32,000Accumulateddepreciation

55,300 -

Estimated useful life 10 years 10 yearsThe company is not subject to tax and its cost of capital is 12%. What is the presentvalue of all the relevant cash flows at time zero? A. (P 54,000) C. (P106,000)B. (P120,000) D. (P124,700)

xxi. Consider a project that requires an initial cash outflow of P500,000 with a life of eightyears and a salvage value of P20,000 upon its retirement. Annual cash inflow beforetax amounts to P100,000 and a tax rate of 30 percent will be applicable. The requiredminimum rate of return for this type of investment is 8 percent. The present value of 1and the annuity of 1, discounted at 8 percent for 8 periods are 0.54 and 5.747,respectively. Salvage value is ignored in computing depreciation. The net presentvalue amounts toA. P 7,560 C. P 17,606B. P 10,050 D. P 20,050

xxii. Zap Manufacturing has an investment opportunity to embark on a project where yearlyrevenues for five years are to be P400,000 and operating costs of P104,800. Theequipment costs P1 million, and straight-line depreciation will be used for book and taxpurposes. No salvage value is expected at the end of the project’s life. The companyhas a 40 percent marginal tax rate and a 10 percent cost of capital. The equipmentmanufacturer has offered a delayed payment plan of P560,500 per year at the end ofthe first and second years. There will be no changes in working capital.The present value of annuity of 1 for 5 periods is 3.7908 at 10 percent. The present values of 1 end of each period at 10 percent are:

Period 1 0.9091Period 2 0.8264Period 3 0.7513Period 4 0.6830Period 5 0.6209

The net present value if the equipment were purchased isA. P (87,977) C. P 1,922B. P (25,310) D. P (61,094)

xxiii. Paz Insurance Company’s management is considering an advertising program thatwould require an initial expenditure of P165,500 and bring in additional sales over thenext five years. The cost of advertising is immediately recognized as expense. Theprojected additional sales revenue in Year 1 is P75,000, with associated expenses ofP25,000. The additional sales revenue and expenses from the advertising program areprojected to increase by 10 percent each year. Paz Insurance Company’s tax rate is 40percent.

The present value of 1 at 10 percent, end of each period:

239

Page 14: MODULE 9 CAPITAL BUDGETING THEORIES: Basic Concepts

Capital Budgeting

Period Present value of 11. 0.909092. 0.826453. 0.751314. 0.683015. 0.62092

The net present value of the advertising program would beA. P 37,064 C. P 29,136B. P(37,064) D. P(29,136)

xxiv.Mario Hernandez plans to buy a haymaker. It costs P175,000 and is expected to last forfive years. He presently hires 6 workers at P10,000 per month for each of the threeharvesting months each year. The equipment would eliminate the need for twoworkers. Hernandez uses straight-line depreciation and projects a salvage value ofP25,000. His tax rate is 25% and opportunity cost of funds is 12.0%. The present valueof 1discounted at 12 percent at the end of 5 periods is 0.56743 and the present valueof an annuity of 1 for 5 periods is 3.60478. Which of the following is true?A. The present value of cash flows in year 5 is P22,710B. NPV is P28,436C. NPV is P15,250D. NPV is P14,186

xxv. Tabucol Aggregates, Inc. plans to replace one of its machines with a new efficient one.The old machine has a net book value of P120,000 with remaining economic life of 4years. This old machine can be sold for P80,000. If the new machine were acquired,the cash operating expenses will be reduced from P240,000 to P160,000 for each of thefour years, the expected economic life of the new machine. The new machine will costTabucol a cash payment to the dealer of P300,000. The company is subject to 32percent tax and for this kind of investment, a marginal cost of capital of 9 percent. Thepresent value of annuity of 1 and the present value of 1 for 4 periods using 9 percentare 3.23972 and 0.70843, respectively.The net present value to be provided by the replacement of the old machine isA. P28,493 C. P46,794B. P15,693 D. P59,594

xxvi.Zambales Mines, Inc. is contemplating the purchase of equipment to exploit a mineraldeposit that is located on land to which the company has mineral rights. Anengineering and cost analysis has been made, and it is expected that the followingcash flows would be associated with opening and operating a mine in the area.

Cost of new equipment and timbers 2,750,000Working capital required 1,000,000Net annual cash receipts* 1,200,000Cost to construct new road in three years 400,000Salvage value of equipment in 4 years 650,000

*Receipts from sales of ore, less out-of-pocket costs for salaries, utilities, insurance,etc.

It is estimated that the mineral deposit would be exhausted after four years of mining.At that point, the working capital would be released for reinvestment elsewhere. Thecompany’s discount rate is 20%.The net present value for the project is:A. P 454,620. C. P(561,553)B. P (79,303). D. P(204,688).

With inflationxxvii. By the end of December 31, 2005, Alay Foundation is considering the purchase of a

copying machine for P80,000. The expected annual cash savings are expected to beP32,000 in the next four years. At the end of the four years, the machine will bediscarded without any salvage value. All the cash savings are stated in number ofpesos at December 31, 2006. The company expected that the inflation rate isconstantly 5 percent each year. Hence, the first year’s cash inflow was adjusted for 5percent inflation. For simplicity, all cash inflows are assumed to be at year-end. The present value at 14 % of 1 for 4 periods is 2.91371. The present value of 1 at endof each period are:

240

Page 15: MODULE 9 CAPITAL BUDGETING THEORIES: Basic Concepts

Capital Budgeting

Period 1 0.87719Period 2 0.76947Period 3 0.67497 Period 4 0.59208

Using the nominal rate of return of 14 percent, the net present value for this machine isA. P12,239 C. P13,419B. P19,670 D. P27,936

xxviii. Perpetual Foundation, Inc., a nonprofit organization, has one of its activities, theproduction of cookies for its snack food store. Several years ago, Perpetual Foundation,Inc. purchased a special cookie-cutting machine. As of December 31, 2006, thismachine will have been used for three years. Management is considering the purchaseof a newer, more efficient machine. If purchased, the new machine would be acquiredon December 31, 2006. Management expects to sell 300,000 dozen cookies in each ofthe next six years. The selling price of the cookies is expected to average P1.15 perdozen.Perpetual Foundation, Inc. has two options: continue to operate the old machine, orsell the old machine and purchase the new machine. No trade-in was offered by theseller of the new machine. The following information has been assembled to helpmanagement decide which option is more desirable.

OldMachine

NewMachine

Original cost of machine atacquisition

P80,000 P120,000

Remaining useful life as of12/31/06

6 years 6 years

Expected annual cash operatingexpenses: Variable cost per dozen P0.38 P0.29 Total fixed costs P21,000 P 11,000Estimated cash value ofmachines: December 31, 2006 P40,000 P120,000 December 31, 2012 P 7,000 P 20,000

Assume all operating revenues and expenses occur at the end of the year.The net advantage in present value of the better alternative is: A. Retain Old Machine, P61,675.B. Buy New Machine, P61,675.C. Retain Old Machine, P16,345.D. Buy New Machine, P16,345.

Profitability indexxxix.The Pambansang Kamao Corporation has to replace its completely damaged boiler

machine with a new one. The old machine has a net book value of P100,000 with zeromarket value; therefore it will give a tax shield, based on 35% tax rate if replaced, byP35,000. The company has a 10 percent cost of capital. Understandably, the newmachine, through a uniform decrease in cash operating costs, will give a positive netpresent value, because this machine will provide an internal rate of return of 12percent.The present values at 10% and 12%, respectively, are:

10% 12%Annuity of 1, 6 periods 4.35526 4.111411 end of 6 periods 0.56447 0.50663

If the machine were to be depreciated using straight-line method for 6 years withoutany salvage value, the estimated profitability index is:A. 1.20B. 1.06C. 1.07D. Cannot be determined from the information

xxx. The Mejicano Company is planning to purchase a piece of equipment that will reduceannual cash expenses over its 5-year useful life by equal amounts. The company willdepreciate the equipment using straight-line method of depreciation based on

241

Page 16: MODULE 9 CAPITAL BUDGETING THEORIES: Basic Concepts

Capital Budgeting

estimated life of 5 years without any salvage value. The company is subject to 40percent tax. The marginal cost of capital for this acquisition is 11.055 percent. Themanagement accountant calculated that the internal rate of return based on theestimated after-tax cash flows is 12.386 percent and a net present value of P10,000.The president, however, wants to know the profitability index before he finally decides.What is the profitability index for this investment?A. 1.011 C. 1.022B. 1.034 D. 1.044

Internal Rate of Returnxxxi.Diamond Company is planning to buy a coin-operated machine costing P400,000. For

book and tax purposes, this machine will be depreciated P80,000 each year for fiveyears. Diamond estimates that this machine will yield an annual inflow, net ofdepreciation and income taxes, of P120,000. Diamond’s desired rate of return on itsinvestments is 12%. At the following discount rates, the NPVs of the investment in thismachine are:

Discount Rate NPV12% +P3,25814% + 1,19716% - 70818% - 2,474

Diamond’s expected IRR on its investment in this machine isA. 3.25% C. 16.00%B. 12.00% D. 15.30%

Required investmentxxxii. Kipling Company has invested in a project that has an eight-year life. It is

expected that the annual cash inflow from the project will be P20,000. Assuming thatthe project has a internal rate of return of 12%, how much was the initial investmentin the project if the present value of annuity of 1 for 8 periods is 4.968 and thepresent value of 1 is 0.404?A. P160,000 C. P 80,800B. P 99,360 D. P 64,640

xxxiii. Katol Company invested in a machine with a useful life of six years and no salvagevalue. The machine was depreciated using the straight-line method. It was expectedto produce annual cash inflow from operations, net of income taxes, of P6,000. Thepresent value of an ordinary annuity of P1 for six periods at 10% is 4.355. The presentvalue of P1 for six periods at 10% is 0.564. Assuming that Katol used a time- adjustedrate of return of 10%, what was the amount of the original investment?A. P10,640 C. P22,750B. P29,510 D. P26,130

xxxiv. The Forest Company is planning to invest in a machine with a useful life of fiveyears and no salvage value. The machine is expected to produce cash flow fromoperations, net of income taxes, of P20,000 in each of the five years. Forest’s expectedrate of return is 10%. Information on present value and future amount factors is asfollows:

P E R I O D1 2 3 4 5

Present value of P1 at 10% .909 .826 .751 .683 .621Present value of an annuity ofP1 at 10%

.909 1.736

2.487

3.170

3.791

Future amount of P1 at 10% 1.100

1.210

1.331

1.464

1.611

Future amount of an annuityof P1 at 10%

1.00 2.100

3.310

4.641

6.105

How much will the machine cost?A. P 32,220 C. P 75,820 B. P 62,100 D. P122,100

Required unit salesxxxv. Paper Products Company is considering a new product that will sell for P100 and has

242

Page 17: MODULE 9 CAPITAL BUDGETING THEORIES: Basic Concepts

Capital Budgeting

a variable cost of P60. Expected volume is 20,000 units. New equipment costingP1,500,000 and having a five-year useful life and no salvage value is needed, and willbe depreciated using the straight-line method. The machine has fixed cash operatingcosts of P200,000 per year. The firm is in the 40 percent tax bracket and has cost ofcapital of 12 percent. The present value of 1, end of five periods is 0.56743; presentvalue of annuity of 1 for 5 periods is 3.60478.How many units per year the firm must sell for the investment to earn 12 percentinternal rate of return?A. 17,338 C. 9,838B. 28,897 D. 12,338

Required selling pricexxxvi. Bughaw Products Company is considering a new product that will sell for P100 and

has a variable cost of P60. Expected sales volume is 20,000 units. New equipmentcosting P1,500,000 with a five-year useful life and no terminal salvage value is needed.The machine will be depreciated using the straight-line method. The machine has cashoperating costs of P200,000 per year. The firm is in the 40 percent tax bracket and hascost of capital of 12 percent. The present value of 1, end of five periods is 0.56743;present value of annuity of 1 for 5 periods is 3.60478.Suppose the 20,000 estimated sales volume is sound, but the price is in doubt, what isthe selling price (rounded to nearest peso) needed to earn a 12 percent internal rate ofreturn?A. P81.00 C. P70.00B. P95.00 D. P90.00

Required CFBTxxxvii. Aloha Co. is considering the purchase of a new ocean-going vessel that could

potentially reduce labor costs of its operation by a considerable margin. The new shipwould cost P500,000 and would be fully depreciated by the straight-line method over10 years. At the end of 10 years, the ship will have no value and will be sunk in somealready polluted harbor. The Aloha Co.’s cost of capital is 12 percent, and its marginaltax rate is 40 percent. If the ship produces equal annual labor cost savings over its 10-year life, how much do the annual savings in labor costs need to be to generate a netpresent value of P0 on the project?Use the following PV: annuity of 1, 10 periods at 12% - 5.6502; end of 10th period –0.32197.A. P 68,492 C. P114,154B. P147,487 D. P 88,492

Required CFATxxxviii. Prudu Company has decided to invest in some new equipment. The equipment will

have a three-year life and will produce a uniform series of cash savings. The netpresent value of the equipment is P1,750, using a discount rate of 8 percent. Theinternal rate of return is 12 percent.Present values at 8% and 12% respectively:

8%: Annuity – 2.5771; end of 3 periods, 0.793812%: Annuity – 2,4018; end of 3 periods, 0.7118

What is the amount of annual cash inflow?A. P 9,980 C. P23,240B. P21,342 D. P12,351

xxxix. An asset is purchased for P120,000. It is expected to provide an additional P28,000of annual net cash inflows. The asset has a 10-year life and an expected salvage valueof P12,000. The hurdle rate is 10%. The present value of an annuity factor of 10% for10 years is 6.1446, and the present value of P1, discounted for 10 years at 10% is0.3855. Given the data provided, the minimum amount of annual cash inflows that wouldprovide the 10% time-adjusted return is approximatelyA. P18,776 C. P24,400B. P26,600 D. P22,535

Required Increase in CFATxl. The following data pertain to Julian Corp. whose management is planning to purchase a

243

Page 18: MODULE 9 CAPITAL BUDGETING THEORIES: Basic Concepts

Capital Budgeting

unit of equipment.1. Economic life of equipment – 8 years.2. Disposal value after 8 years – Zero.3. Estimated net annual cash inflows for each of the 8 years – P81,000.4. Time-adjusted internal rate of return – 14%5. Cost of capital of Bayan Muna – 16%6. The table of present values of P1 received annually for 8 years has these factors:

at 14% = 4.639, at 16% = 4.3447. Depreciation is approximately P46,970 annually.

Find the required increase in annual cash inflows in order to have the time-adjustedrate of return approximately equal the cost of capital. A. P6,501 C. P4,344B. P5,501 D. P5,871

Required CFAT for a certain yearxli. A company is considering putting up P50,000 in a three-year project. The company’s

expected rate of return is 12%. The present value of P1.00 at 12% for one year is0.893, for two years is 0.797, and for three years is 0.712. The cash flow, net ofincome taxes will be P18,000 (present value of P16,074) for the first year and P22,000(present value of P17,534) for the second year. Assuming that the rate of return isexactly 12%, the cash flow, net of income taxes, for the third year would beA. P23,022 C. P10,000B. P 7,120 D. P16,392

Required salvage valuexlii. The Caravan Company is contemplating to purchase a machine that costs P800,000.

The machine is expected to last for 5 years with a salvage value of P50,000 at the endof the fifth year. If the machine were purchased, before-tax annual cash savings onoperating expenses will be realized. Caravan Company will depreciate the machineusing straight-line depreciation for 5 years, with the salvage value considered in thecomputation.The company has a 12 percent cost of capital and is subject to 40 percent tax rate.The present values using 12 percent are:

Annuity of 1 for 5 periods 3.60478Present value of 1, end of 5 periods 0.56743

The initial analysis indicated a net present value of P7,003. You believe the estimatedbefore-tax cash savings are fairly determined but you are in doubt of the expectedsalvage value of the machine. How much is the estimated salvage value required if the investment has to yield an IRRof 12 percent?A. P41,800 C. P25,100B. P24,900 D. P44,600

Required value of intangible benefitsxliii. Solidum Company is investigating the purchase of a piece of automated equipment

that will save P100,000 each year in direct labor and inventory carrying costs. Thisequipment costs P750,000 and is expected to have a 10-year useful life with nosalvage value. The company requires a minimum 15% return on all equipmentpurchases. Management anticipates that this equipment will provide intangible benefitssuch as greater flexibility and higher quality output.

The PV of annuity of 1, 15% for 10 periods 5.01877The PV of 1, end 10 period 0.24718

What peso value per year would these intangible benefits have to have in order tomake the equipment an acceptable investment?A. P248,123 C. P 61,331B. P 49,440 D. P 55,000

xliv. Altas, Inc., is considering investing in automated equipment with a ten-year usefullife. Managers at Altas have estimated the cash flows associated with the tangiblecosts and benefits of automation, but have been unable to estimate the cash flowsassociated with the intangible benefits. Using the company’s 10% discount rate, thenet present value of the cash flows associated with just the tangible costs andbenefits is a negative P184,350. The present value of annuity of 1 at 10 percent for

244

Page 19: MODULE 9 CAPITAL BUDGETING THEORIES: Basic Concepts

Capital Budgeting

ten years is 6.145 while the present value of 1 is 0.386. How large would the annualnet cash inflows from the intangible benefits have to be to make this a financiallyacceptable investment?A. P18,435. C. P35,000.B. P30,000. D. P37,236.

Indifference Pointxlv. Moon Company uses a 10% discount rate and the total cost approach to capital

budgeting analysis. Both alternatives are Akda Investments which has a marginal costof capital of 12 percent is evaluating two mutually exclusive projects (X and Y), whichhave the following projections:

PROJECT X PROJECT YInvestment P48,000 P83,225After-tax cashinflow

12,000 15,200

Asset life 6 years 10 yearsThe indifference point for the two projects isA. 12.64% C. 12.00%B. 16.01% D. 19.33%

xlvi. Silky Products is considering two pieces of machinery. The first machine costs P50,000more than the second machine. During the two-year life of these two alternatives, thefirst machine has a P155,000 more cash flow in year one and a P110,000 less cash flowin year two than the seconds machine. All cash flows occur at year-end. The presentvalue of 1 at 15 percent end of 1 period and 2 periods are 0.86957 and, 0.75614,respectively. The present value of 1 at 8 percent end of period 1 is 0.92593, and Period2 is 0.85734.At what discount rate would Machine 1 be equally acceptable as machine 2’s?A. 9% C. 11%B. 10% D. 12%

Decision Rule – Independent Projectsxlvii. Sylvia Products is considering two types of machinery. The first machine costs P50,000

more than the second machine. During the two-year life of these two alternatives, thefirst machine has a P155,000 more cash flow in year one and a P110,000 less cash flowin year two than the seconds machine. All cash flows occur at year-end. The presentvalue of 1 at 15 percent end of 1 period and 2 periods are 0.86957 and, 0.75614,respectively. The present value of 1 at 8 percent end of period 1 is 0.92593, and Period2 is 0.85734.Which machine should be purchased if the relevant discount rates are 15 percent and 8percent, respectively?

15% Discount 8% DiscountA. Machine 1 Machine 1B. Machine 2 Machine 2C. Machine 1 Machine 2D. Machine 2 Machine 1

ComprehensivePayback, NPV, ARRQuestion Nos. 71 through 73 are based on the following:Cayco Medical Center is considering purchasing an ultrasound machine for P950,000. Themachine has a 10 – year life and an estimated salvage value of P55,000. Installation costsand freight charges will be P24,200 and P800, respectively. Newman uses straight-linedepreciation.

The medical center estimates that the machine will be used five times a week with theaverage charges to the patient for ultrasound of P800. There are P10 in medical suppliesand P40 of technician costs for each procedure performed using the machine. The presentvalue of an annuity of 1 for 10 years at 9% is 6.418 while the present value of 1 for 10years at 9% is 0.42241

xlviii. The cash payback period is:

245

Page 20: MODULE 9 CAPITAL BUDGETING THEORIES: Basic Concepts

Capital Budgeting

A. 3.0 years C. 5.0 yearsB. 4.5 years D. 6.0 years

xlix. The project is expected to generate net present value of:A. P276,510 C. P331,510B. P299,743 D. P253,277

l. What is the accounting rate of return provided by the project?A. 20.0 percent C. 11.2 percentB. 10.6 percent D. 38.0 percent

NPV, CFAT, Maximum lost unit salesQuestion Nos. 75 through 77 are based on the following:Kabalikat Company has the opportunity to introduce a new product. Kabalikat expects theproduct to sell for P75 with variable cost per unit of P50. The annual fixed costs, excludingthe amount of depreciation is P4,500,000. The company expects to sell 300,000 units. Toproduce the new product line, the company needs to purchase a new machine that costsP6,000,000. The new machine is expected to last for four years with a very negligiblesalvage value. The company has a policy of depreciating its machine for both book andtax purposes for four years. The company has a marginal cost of capital of 13.75 percentand is subject to tax rate of 40 percent.

li. The amount of annual after-tax cash flows is:A. P2,400,000 C. P 900,000B. P3,000,000 D. P1,500,000

lii. The machine’s net present value is:A. P2,786,100 C. P1,028,900B. P 928,500 D. P 150,270

liii. Assuming that some of the 300,000 units that are expected as sales would be to groupof customers who currently buy K-Z, another product of Kabalikat Company. ThisProduct K-Z sells for P35 with variable cost of P20. How many units of K-Z canKabalikat afford to lose before the purchase of the new machine becomes unattractive?A. 39,000 units C. 16,714 unitsB. 23,400 units D. 10,029 units

ARR, NPV, PI, PaybackQuestions 1 through 4 will be based on the following data:The management of Arleen Corporation is considering the purchase of a new machinecosting P400,000. The company’s desired rate of return is 10%. The present value of P1 atcompound interest of 10% for 1 through 5 years are 0.909, 0.826, 0.751, 0.683, and0.621, respectively, and the present value of annuity of 1 for 5 periods at 10 percent is3.79. In addition to the foregoing information, use the following data in determining theacceptability in this situation:

Year Income from Operations Net Cash Flow1 P100,000 P180,0002 40,000 120,0003 20,000 100,0004 10,000 90,0005 10,000 90,000

liv. The average rate of return for this investment is:A. 18 percent C. 58 percentB. 6 percent D. 10 percent

lv. The net present value for this investment is:A. Positive P 36,400 C. Negative P 99,600B. Positive P 55,200 D. Negative P126,800

lvi. The present value index for this investment is:A. 0.88 C. 1.14

246

Page 21: MODULE 9 CAPITAL BUDGETING THEORIES: Basic Concepts

Capital Budgeting

B. 1.45 D. 0.70

lvii. The cash payback period for this investment is:A. 4 years C. 20 yearsB. 5 years D. 3 years

Payback, NPV, ARR, IRRUse the following information for questions 67 - 70Pillo Company is considering two capital investment proposals.

Estimates regarding each project are provided below:

Project MA Project PAInitial investment P2000,000 P300,000Annual net income 10,000 21,000Net annual cash inflow 50,000 71,000Estimated useful life 5 years 6 yearsSalvage value -0- -0-

The company requires a 10% rate of return on all new investments.

Present Value of an Annuity of 1

Period 9% 10% 11% 12%5 3.890 3.791 3.696 3.6056 4.486 4.355 4.231 4.111

lviii. The cash payback period for Project MA isA. 20 years C. 5 yearsB. 10 years D. 4 years

lix. The net present value for Project PA isA. P309,204 C. P 50,000B. P 91,456 D. P 9,205

lx. The annual rate of return for Project MA isA. 5% C. 25%B. 10% D. 50%

lxi. The internal rate of return for Project PA is closest toA. 10% C. 12%B. 11% D. none of these

Depreciation tax shield, CCFAT, Payback, NPV, IRRQuestion Nos. 86 through 90 are based on the following:Consider a project that requires cash outflow of P50,000 with a life of eight years and asalvage value of P2,000. Annual cash inflow amounts to P10,000 assuming a tax rate of30% and a required rate of return of 8%. Salvage value is ignored in computingdepreciation.

lxii. Annual depreciation tax shield amounts toA. P1,875 C. P8,875B. P7,000 D. P10,000

lxiii. Annual cash flow after tax amounts toA. P 1,875 C. P 8,875B. P 7,000 D. P10,000

lxiv. Payback amounts toA. 5.0 years C. 6.0 yearsB. 5.6 years D. 6.6 years

lxv. Net present value amounts to

247

Page 22: MODULE 9 CAPITAL BUDGETING THEORIES: Basic Concepts

Capital Budgeting

A. P 756 C. P1,756B. P1,005 D. P2,005

lxvi. Internal rate of return on this project is approximatelA. 8.0% C. 9.0%B. 8.5% D. 9.5%

CFAT, NPV, IRRQuestions 46 rough 51 are based on the following:Home’s Pizza’s, Inc., operates pizza shops in several cities. One of the company’s mostprofitable shops is located adjacent to the large CPA review center in Manila. A smallbakery next to the shop has just gone out of business, and Home’s Pizzas has anopportunity to lease the vacated space for P18,000 per year under a 15-year lease.Home’s management is considering two ways in which the available space might be used.

Alternative 1. The pizza shop in this location is currently selling 40,000 pizzas per year.Management is confident that sales could be increased by 75% by taking out the wallbetween the pizza shop and the vacant space and expanding the pizza outlet. Costs forremodeling and for new equipment would be P550,000. Management estimates that 20%of the new sales would be small pizzas, 50% would be medium pizzas, and 30% would belarge pizzas. Selling prices and costs for ingredients for the three sizes of pizzas follow (perpizza):

Selling Price Cost of IngredientsSmall P 6.70 P1.30Medium 8.90 2.40Large 11.00 3.10

An additional P7,500 of working capital would be needed to carry the larger volume ofbusiness. This working capital would be released at the end of the lease term. Theequipment would have a salvage value of P30,000 in 15 years, when the lease ends.Alternative 2. Home’s sales manager feels that the company needs to diversify itsoperations. He has suggested that an opening be cut in the wall between the pizza shopand the vacant space and that video games be placed in the space, along with a smallsnack bar. Costs for remodeling and for the snack bar facilities would be P290,000. Thegames would be leased from a large distributor of such equipment. The distributor hasstated that based on the use of game centers elsewhere, Home’s could expect about26,000 people to use the center each year and to spend an average of P5 each on themachines. In addition, it is estimated that the snack bar would provide a net cash inflow ofP15,000 per year. An investment of P4,000 in working capital would be needed. Thisworking capital investment would be released at the end of the lease term. The snack barequipment would have a salvage value of about P12,000 in 15 years.

Home’s management is unsure which alternative to select and has asked you to help inmaking the decision. You have gathered the following information relating to added coststhat would be incurred each year under the two alternatives:

Expand the PizzaShop

Install the GameCenter

Rent- building space P18,000 P18,000Rent- video games --- 30,000Salaries 54,000 17,000Utilities 13,200 5,400Insurance and other 7,800 9,600

The company is currently using a 16 percent minimum acceptable rate of return for itscapital investment. The present value of annuity of 1 at 16 percent for 15 periods is 5.575and end of 15 periods is 0.108. The company is not liable to pay income taxes.

lxvii. The incremental expected annual cash inflows from Alternative 1 is:A. P 90,000 C. P100,200B. P108,000 D. P201,000

248

Page 23: MODULE 9 CAPITAL BUDGETING THEORIES: Basic Concepts

Capital Budgeting

lxviii. The incremental expected annual cash inflows from Alternative 2 is:A. P 17,000 C. P 59,600B. P 65,000 D. P145,000

lxix. The net present value for Alternative 1 is:A. P48,650 C. P45,000B. P47,840 D. P32,500.

lxx. The net present value for Alternative 2 is:A. P21,021 C. P68,375B. P70,103 D. P12,807

lxxi. Assume that the company decides to accept alternative 2. At the end of the first year,the company finds that only 21,000 people used the game center during the year (eachperson spent P5 on games). Also, the snack bar provided a net cash inflow of onlyP13,000. In light of this information, what is the net present value for alternative 2?A. P(80,422) C. P(82,150)B. P(76,422) D. P(80,854)

lxxii. The sales manager has suggested that an advertising program be initiated to drawanother 5,000 people into the game center each year. Assuming that another 5,000people can be attracted into the center and that the snack bar receipts increase to thelevel originally estimated, how much can be spent on advertising each year and stillallow the game center to provide a 16% rate of return?A. P70,103.00 C. P58,953.00B. P 4,673.53 D. P12,574.53

Net Income, CFBT, ARR, Payback PeriodQuestions 52 through 56 are based on the following information:Pinewood Craft Company is considering the purchase of two different items of equipment,as described below:

Machine A. A compacting machine has just come onto the market that would permitPinewood Craft Company to compress sawdust into various shelving products. At presentthe sawdust is disposed of as a waste product. The following information is available onthe machine:

a. The machine would cost P420,000 and would have a 10% salvage value at the endof its 12-year useful life. The company uses straight-line depreciation and considerssalvage value in computing depreciation deductions.

b. The shelving products manufactured from use of the machine would generaterevenues of P300,000 per year. Variable manufacturing costs would be 20% ofsales.

c. Fixed expenses associated with the new shelving products would be (per year):advertising, P40,000; salaries, P110,000; utilities, P5,200; and insurance, P800.

Machine B. A second machine has come onto the market that would allow PinewoodCraft Company to automate a sanding process that is now done largely by hand. Thefollowing information is available:

a. The new sanding machine would cost P234,000 and would have no salvage value atthe end of its 13-year useful life. The company would use straight-line depreciationon the new machine.

b. Several old pieces of sanding equipment that are fully depreciated would bedisposed of at a scrap value of P9,000.

c. The new sanding machine would provide substantial annual savings in cashoperating costs. It would require an operator at an annual salary of P16,350 andP5,400 in annual maintenance costs. The current, hand-operated sandingprocedure costs the company P78,000 per year in total.

Pinewood Craft Company requires a simple rate of return of 15% on all equipmentpurchases. Also, the company will not purchase equipment unless the equipment has apayback period of 4.0 years or less.

249

Page 24: MODULE 9 CAPITAL BUDGETING THEORIES: Basic Concepts

Capital Budgeting

(In all the following questions, please ignore income tax effect)

lxxiii. The expected income each year from the new shelving products (Machine A) is:A. P 52,500 C. P 84,000B. P240,000 D. P 92,500

lxxiv. The annual savings in cost if Machine B is purchased isA. P56,250 C. P38,250B. P43,250 D. P21,750

lxxv.The simple rate (%) of return for Machine A is:A. 12.5 percent C. 25.0 percentB. 20.0 percent D. 18.0 percent

lxxvi. The simple rate of return for Machine B is:A. 16.3 percent C. 25.0 percentB. 17.0 percent D. 34.0 percent

lxxvii. The payback period for Machine A is:A. 3.0 years C. 5.0 years B. 4.5 years D. 7.5 years

lxxviii. The payback period for Machine B is:A. 4.0 years. C. 6.1 years.B. 4.2 years. D. 5.9 years.

Net Investment, CFBT, Tax Benefits, NPV, Depreciation Tax Shield, Question Nos. 58 through 63 are based on the following:Turkey Company’s average production of valve stems over the past three years has been80,000 units each year. Expectations are that this volume will remain constant over thenext four years. Cost records indicate that unit product costs for the valve stem over thelast several years have been as follows:

Direct materials P 3.60Direct labor 3.90Variable manufacturing overhead 1.50Fixed manufacturing overhead* 9.00Unit product cost P18.00

*Depreciation of tools (that must now be replaced) accounts for one-third of the fixedoverhead. The balance is for other fixed overhead costs of the factor that require cashexpenditures.

If the specialized tools are purchased, they will cost P2,500,000 and will have a disposalvalue of P100,000 at the end of their four-year useful life. Turkey Company has a 30% taxrate, and management requires a 12% after-tax return on investment. Straight-linedepreciation would be used for financial reporting purposes, but for the tax purposes, thefollowing variable depreciation each year will be used.

Year 1 P 832,500Year 2 1,112,500Year 3 370,000Year 4 185,000

The sales representative for the manufacturer of the specialized tools has stated, “Thenew tools will allow direct labor and variable overhead to be reduced by P1.60 per unit.”Data from another company using identical tools and experiencing similar operatingconditions, except that annual production generally averages 100,000 units, confirms thedirect labor and variable overhead cost savings. However, the other company indicatesthat it experienced an increase in raw material cost due to the higher quality of materialthat had to be used with the new tools. The other company indicates that its unit productcosts have been as follows:

250

Page 25: MODULE 9 CAPITAL BUDGETING THEORIES: Basic Concepts

Capital Budgeting

Direct materials P 4.50Direct labor 3.00Variable manufacturing overhead 0.80Fixed manufacturing overhead 10.80Unit product cost P19.10

Referring to the figures above, the production manager stated, “These numbers look greatuntil you consider the difference in volume. Even with the reduction in labor and variableoverhead cost, I’ll bet our total unit cost figure would increase to over P20 with the newtools.”

Although the old tools being used by Turkey Company are now fully depreciated, they havea salvage value of P45,000. These tools will be sold if the new tools are purchased;however if the new tools are not purchased, then the old tools will be retained as standbyequipment. Turkey Company’s accounting department has confirmed that total fixedmanufacturing overhead costs, other than depreciation, will not change regardless of thedecision made concerning the valve stems. However, the accounting department hasestimated that working capital needs will increase by P60,000 if the new tools arepurchased due to the higher quality of material required in the manufacture of the valvestems.

The present values of 1 at the end of each period using 12 percent are:Period 1 0.89286Period 2 0.79719Period 3 0.71178Period 4 0.63552PV of annuity of 1, 4 periods 3.03735

lxxix. The net investment in new tools amounted to:A. P1,873,300. C. P2,528,500.B. P2,515,000. D. P2,546.500.

lxxx.How much annual cost savings will be generated if the Turkey Company purchases thenew tools?A. P 128,000 C. P 936,000B. P 216,000 D. P1,008,000

lxxxi. The present value of tax benefits expected from the use of the new machine toolsis:A. P 603,333 C. P1,407,777B. P 804,444 D. P2,011,111

lxxxii. The present value of the salvage value of the new tools to be received at the end offourth year isA. P 63,552. C. P 44,486.B. P 19,065. D. P212,615.

lxxxiii. Using the minimum acceptable rate of return of 12 percent, the net present value ofthe investment in new tools isA. P108,913. C. P147,073.B. P127,979. D. P166,139.

lxxxiv. The net advantage of the use of declining method of depreciation instead ofstraight-line method isA. P 33,830. C. P112,767.B. P 56,610. D. P147,731.

Net Investment, CFAT, Depreciation tax shield, NPVQuestion Nos. 77 through 82 are based on the following:Franzen Company manufactures three different models of paper shredders including thewaste container, which serves as the base. While the shredder heads are different for all

251

Page 26: MODULE 9 CAPITAL BUDGETING THEORIES: Basic Concepts

Capital Budgeting

three models, the waste container is the same. The number of waste containers thatFranzen will need during the next five years is estimated as follows:

2007 50,0002008 50,0002009 52,0002010 55,0002011 55,000

The equipment used to manufacture the waste container must be replaced because it isbroken and cannot be repaired. The new equipment would have a purchase price ofP945,000 with terms 2/10, n/30; the company’s policy is to take all purchase discounts.The freight on the equipment would be P11,000, and installation costs would totalP22,900. The equipment would be purchased in December 2006 and placed into serviceon January 1, 2007. It would have a five-year economic life and would have the followingdepreciation. The equipment is expected to have a salvage value of P12,000 at the end ofits economic life in 2011. The new equipment would be more efficient than the oldequipment, resulting in a 25 percent reduction in both direct material and variableoverhead. The savings in direct material would result in an additional one-time decreasein working capital requirements of P2,500, resulting from a reduction in direct materialinventories. This working capital reduction would be recognized at the time of equipmentacquisition.

The old equipment is fully depreciated and is not included in the fixed overhead. The oldequipment from the plant can be sold for a salvage amount of P1,500. Rather thanreplace the equipment, one of Franzen’s production managers has suggested that thewaste containers be purchased. One supplier has quoted a price of P27 per container.This price is P8 less than Franzen’s current manufacturing cost, which is presented below.

Direct materials P10Direct labor 8Variable overhead 6Fixed overhead: Supervision P2 Facilities 5 General 4 11Total unit cost P35

Franzen uses a plantwide fixed overhead rate in its operations. If the waste containers arepurchase outside, the salary and benfits of one supervisor, included in fixed overhead ofP45,000 would be eliminated. There would be no other changes in the other cash andnoncash items included in fixed overhead except depreciation on the new equipment.

The new equipment will be depreciated according to the following declining amounts:Year Depreciation2007 P319,9682008 426,7202009 142,1762010 71,1362011 0

Franzen is subject to a 40 percent tax rate. Management assumes that all cash flowsoccur at the end of the year and uses a 12 percent after-tax discount rate.

lxxxv. The initial net cash outflows if the company decides to continue making the wastecontainers is:A. P 956,600 C. P 978,900B. P 975,500 D. P1,455,613

lxxxvi. The total after-tax cash outflows, excluding the initial cash outflows, if the newequipment is purchased are:A. P 956,600 C. P2,918,300B. P2,887,800 (defective) D. P3,279,000

lxxxvii. The present value of the total depreciation shield is:

252

Page 27: MODULE 9 CAPITAL BUDGETING THEORIES: Basic Concepts

Capital Budgeting

A. P308,920 C. P307,826B. P313,500 D. P321,303

lxxxviii. The total relevant after-tax costs to buy the waste containers are:A. P2,829,240 C. P4,243,500 (defectiveB. P3,039,662 D. P7,074,000

lxxxix. What is the net present value of the purchase alternative?A. P3,039,662 (defective) C. P2,083,062B. P2,730,742 D. P2,718,359

xc. What is the net present value of the make alternative?A. P2,036,603 C. P2,996,603B. P3,039,662 D. P2,993,203 (defective)

ANSWER EXPLANATIONS

253

Page 28: MODULE 9 CAPITAL BUDGETING THEORIES: Basic Concepts

i . Answer: BInitial amount of investment 160,000Less Cash inflow (decrease in outflow) at period 0: MV of old equipment 80,000 Tax benefits on loss on sales (20,000 x .4) 8,000 88,000Net investment 72,000

ii . Answer: DATCF = Net investment ÷ Payback periodATCF (840,000 ÷ 3.326) 252,555Net income (252,555 – 140,000) 112,555Before-tax income (112,555 ÷ 0.60) 187,592Before-tax savings (187,592 + 140,000) 327.592

The computation of after-tax cash flows, given the amount of investment and internal rate of return or PV of annuity of 1discounted at IRR is the reverse of the computation of payback period. Remember that the payback method, though anondiscounted technique, is closely related to internal rate of return because the payback period is exactly the presentvalue of annuity of 1 if they are discounted using the internal rate of return.

iii . Answer: AAnnual savings on expenses P50,000Less: Additional depreciation (40,000 – 25,000) 15,000 Additional taxable income 35,000 Additional tax (35,000 x 40%) P14,000

Additional depreciation can be easily calculated by subtracting the book value of the old machine from the cost of newmachine and then the difference divided by the useful life (160,000 – 100,000) ÷ 4 = 15,000.

iv 10. Answer: BYearSYDStraight LineDifferencePresent Value12,000,0001,200,000800,000

727,28021,600,0001,200,000400,000330,56031,200,0001,200,000 -04 800,0001,200,000(400,000) (273,200)5400,0001,200,000(800,000) (496,720)Total present value of difference in depreciation287,920Tax Rate40%Present value

of net advantage115,168v . Answer: B

SYDSLDifferencePresent Value1 150,00090,00060,00053,5682 120,00090,00030,00023,916390,00090,000-04 60,00090,000(30,000)(19,066)5 30,00090,000(60,000)(34,046)Total of present values ofdepreciation24,372Tax rate40%Present value of net advantage 9,749SYD method provides a higher present value ontax benefits because of less amount of tax during year 1 & 2. In year 4 and 5, the use of SYD requires higher taxes buttheir equivalent present values are lower already.

vi . Answer: DAnnual cost savings 90,000Less depreciation (432,000 ÷ 12) 36,000Annual income 54,000Simple Rate of Return: 54,000 ÷ 432,000 12.5 %

vii . Answer: AThe useful life of the project can be calculated by using the computational pattern for Accounting Rate of Return:

Net investment 106,700Divide by Depreciation expense

CFAT 20,000Less: Net income (106,700 x 5%) 14,665 5,335

Average life (in years) 7.28* 10% ARR based on average investment = 5% ARR based on initial investment

viii . Answer: BARR = Average annual net income ÷ Average Investment Annual after-tax cash flow 40,000Less Depreciation 20,000Net Income 20,000Divide by Average Investment (200,000 + 180,000)/2 190,000ARR: 10.5%

The problem asked for the average accounting rate of return for the first year of asset’s life.

ix . Answer: DThe average (accounting) rate of return is determined by dividing the annual after-tax net income by the average cost ofthe investment, (beginning book value + ending book value)/2.

After tax income (P7,200 - (P7,200 x 30%)) P 5,040Average investment: (P66,000 + 16,000) ÷ 2 P41,000Accounting rate of return: P5,040/P41,000) 12.3%

Page 29: MODULE 9 CAPITAL BUDGETING THEORIES: Basic Concepts

x . Answer: A (ATCF – Depreciation) ÷ Initial investment = Accounting Rate of ReturnLet X = Initial investment(66,000 – 0.10X) ÷ X = 0.12 66,000 - .10X = .12X .22X = 66,000 X = 300,000

xi . Answer: ANet Income: = 66,000 - .10XAAR = NI/ Investment.12 = (66,000 - .10X) / X.12X = 66,000 - .10X.22 X = 66,000 X = 300,000

xii . Answer: DNet Income (280,000 x 15%) 42,000Add back depreciation 35,000ATCF 77,000

xiii . Answer: BPayback period = Initial amount of investment ÷ Annual after-tax cash flows

P35,000 ÷ P5,000 = 7 years

xiv . Answer: BNet investment 50,000Divide by CFAT (10,000 x 0.7) ÷ (50,000 ÷ 8 x 0.3) 8,875Payback period 5.6 years

xv . Answer: DCumulative cash flows end of Year 1 (450,000) – 254,520 (195,480)Discounted cash flow for Year 2 173,460Cumulative cash flows, end of Year 2 ( 22,020)Break-even time 2 + (22,020 ÷ 105,140) 2.21 years

xvi . Answer: DCost of the new machine 400,000Salvage value of old machine at period zero 60,000Net investment (Outflows) 340,000Divide by cash flow after tax 90,000Payback period 3.78 years

xvii . Answer: B Cash InflowUnrecovered OutflowOutflows(4,500,000)First year900,000(3,600,000)Second year1,200,000(2,400,000)Third

year1,500,000( 900,000)Fourth year 900,0000Payback Period: At the end of 4 periods, the initial outflows are fully recovered.Note to the CPA Candidates: A modified question for this problem is to compute the Present Value of the net advantageof using sum-of-the-years’ digits of depreciation instead of straight-line method.

xviii . Answer: CCash inflowsInvestmentPeriod 0(99,300)Period 1 (75,000 – 25,000) x .6 30,000(69,300)Period 2 ( 30,000 x 1.10)

33,000(36,300)Period 3 (33,000 x 1.10) 36,300 -0-At the end of the third year, investment is fully recovered.The net investment of 99,300 is net of tax benefit, (165,500 x .6)

xix . Answer: CBefore-tax cash flow = 40,000 + 35,000 75,000Payback period: 300,000 ÷ 75,000 4 years

xx . Answer: CThere are two cash flows at time zero: P120,000 outflow and P14,000 inflow.Net cash outflow (120,000 – 14,000) = 106,000

xxi . Answer: CComputation of Cash Flow After-tax

CFBT 100,000 x 0.7 70,000Depreciation tax shield 62,500 x 0.3 18,750CFAT 88,750

Computation of Net Present Value:

Page 30: MODULE 9 CAPITAL BUDGETING THEORIES: Basic Concepts

PV of ATCF: 88,750 x 5.747 510,046PV of After-tax Salvage Value: 20,000 x 0.70 x 0.54 7,560Total 517,606Investment 500,000Net Present Value 17,606

The problem assumed that the salvage value is ignored in the computation of annual depreciation so that the annualcash flows will be greater. The problem did not include among the choices the assumption that salvage value will bededucted from the cost in computing the amount of annual depreciation.

xxii . Answer: BAnnual revenues 400,000Less cash operating costs 104,800Cash flow before tax 295,200Less Depreciation (1M ÷ 5) 200,000Income before tax 95,200Less income tax (40%) 28,080Net income 57,120Add back depreciation 200,000ATCF 257,120PV of ATCF, n=5; k=10% 257,120 x 3.7908 974,690Investment 1,000,000Negative Net Present Value ( 25,310)

The manner of financing the project is not considered in the analysis of capital investment. Investment must beseparate from financing. It is a normally committed error in the application of capital budgeting techniques wherefinancing strategy is considered. The explicit or implicit cost of financing the project is taken care of the discountingprocess.

xxiii . Answer: APresent value of cash returns: (30,000 x 0.90909) x 5 periods 136,364Net investment 99,300Net present value 37,064

Note: Because the constant growth rate and the discount rate are both 10%, the present value for each period isconstant.

xxiv . Answer: BSavings (2 workers, each P10,000 for 3 months) 2 x P10,000 x 3 P60,000Depreciation (175,000 – 25,000) ÷ 5 years P30,000After-tax cash savings: (60,000 x 0.75) + (30,000 x 0.25) P52,500Present value of after-tax cash savings (52,500 x 3.60478) P189,250Present value of Salvage Value (25,000 x 0.56743) 14,186Total 203,436Investment 175,000Net Present Value P 28,436

xxv . Answer: BComputation of net investment:

Cash purchase price 300,000Less: MV of old machine 80,000 Tax shield on loss on sale (40,000 x 0.32) 12,800 92,800Net investment 207,200

Annual cash savings before tax (240,000 – 160,000) 80,000Additional depreciation (300,000 – 120,000) ÷ 4 45,000Additional taxable income 35,000Less Additional tax (35,000 x 0.32) 11,200Net income 23,800Add back depreciation 45,000After-tax cash flow 68,800Alternative computation for ATCF:

(80,000 x 0.68) + (45,000 x 0.32) 68,800Present value of ATCF (68,800 x 3.23972) 222,893Investment 207,200Net Present Value 15,693

xxvi . Answer: BPV of annual cash receipts 1,200,000 x 2.58872 3,106,463PV of salvage value 650,000 x 0.48225 313,462PV of return of working capital 1,000,000 x 0.48225 482,250Cost of new equipment and timbers (2,750,000)

Page 31: MODULE 9 CAPITAL BUDGETING THEORIES: Basic Concepts

Working capital (1,000,000)PV of cost of construction of road 400,000 x .5787 ( 231,480)Negative net present value (79,303)

xxvii . Answer: BPeriodNominal Cash SavingsPV FactorPresent Value132,000 0.8779028,070.08232,000 x 1.0533,600

0.7694725,854.19332,000 x 1.05235,280 0.6749723,812.94432,000 x 1.05337,0440.5920821,933.01Total99,670.22Investment80,000.00NPV19,670.22Note that all the annual cash inflows are adjusted

by one period.

xxviii . Answer: BThe solution used total analysis approach in computing present value.Retain the Old Machine:Present value of annual cash outlay CFAT (300,000 x P0.38) + P21,000 = P135,000 PVCFAT (135,000 x 3.6847) P497,435Present value of salvage value (7,000 x 0.41044) ( 2,873) Total P494,562

Buy New machine:Present Value of Annual cash outlay CFAT (300,000 x P0.29) + P11,000 = P98,000 PVCFAT P98,000 x 3.6847) P361,100 Salvage value of new machine, end of 6 years(P20,000 x 0.41044) ( 8,209) Investment in new machine (120,000 – 40,000) 80,000 Total P432,891

xxix . Answer: BThe purpose of profitability index is to compare two projects’ profitability by reducing the present value per 1 peso ofinvestment. Therefore, the ratio of 4.35526 @ 10% to 4.11141 @ 12% indicated the profitability index. Profitability index: 4.35526/4.11141 = 1.06

xxx . Answer: BPV of annuity of 1 at IRR ∑(1 ÷ 1.12386)5 3.57057PV of annuity of 1 at MCC ∑(1 ÷ 1.11055)5 3.69079After-tax cash flows 10,000 ÷ (3.69079 – 3.57057) 83,180.84Investment: 83,180.84 x 3.57057 297,000Profitability index (297,000 + 10,000) ÷ 297,000 1.034A shorter calculation of the Profitability Index can be made by:3.69079 ÷ 3.57057 = 1.034

xxxi . Answer: DIn discounting the annual cash inflow by the IRR, the NPV = P0The net present value of ZERO is 14% and 16%. For better time management, the candidate is expected not to dodetailed calculation of finding out the exact rate.The use of interpolation indicated that the IRR is 15.3%:

Discount RateNet Present Value0.141,197IRR00.16-708(0.14 – IRR) ÷ (0.14 – 0.16) = 1,197 ÷ ( 1,197 + 708)(0.14 – IRR) ÷ -.02 = 1,197 ÷ 1905(0.14 – IRR) ÷ - .02 = 0.628(0.14 – IRR) = 0.628 x -0.020.14 – IRR = 0. 013

IRR = 0.153 or 15.30%Note: Since at the IRR, NPV is zero, the answer can only be between 14% & 16%, since only one of the choices, satisfythe criteria, the answer is (D).

xxxii . Answer: BThe payback period that corresponds to the project’s internal rate of return of 12 percent is 4.968. Therefore, theamount of investment must equal the product of the payback period and the net cash flows:Investment: (4.968 x 20,000) = P99,360

xxxiii . Answer: DThe amount of investment: the PV of annuity at IRR

4.355 x 6,000 = 26,130

xxxiv . Answer: CPresent value of cash inflows equals amount of investment at 10% IRR.

P20,000 x 3.791 = P75,820

Page 32: MODULE 9 CAPITAL BUDGETING THEORIES: Basic Concepts

xxxv . Answer: AATCF: P1,500,000/3.60472 416,121Depreciation 300,000Net income: 416,121 – 300,000 116,121Before-tax income: 116,121/0.60 193,535Fixed costs 500,000Contribution margin: 193,535 + 500,000 693,535Unit sales 693,535 ÷ (100 - 60) 17,338

xxxvi . Answer: BContribution margin (per No. 23) 693,535Divide by sales volume ÷ 20,000Contribution margin per unit P34.68Add variable cost per unit 60.00Selling price per unit P94.68

Alternative Solution:Cash inflow before tax based on present price: (20,000 x 40) – 200,000 600,000After-tax cash inflow (600,000 x 0.6) + (300,000 x 0.4) 480,000Present value of ATCF (480,000 x 3.60478) 1,730,294Investment 1,500,000Net present value (present price) 230,294Annual excess ATCF due to excess price (230,294 ÷ 3.60478) 63,885Before-tax excess cash inflow (63,885 ÷ 0.6) 106,475Excess selling price: 106,475 ÷ 20,000 5.32Reduced selling price to achieve IRR of 12% (100 – 5.32) 94.68

xxxvii. Answer: CAnnual after-tax cash flow 500,000/5.6502 88,492Depreciation 500,000/10 50,000Net income 38,492Income before tax 38,492/0.6 64,154Depreciation 50,000Cash savings before tax: 64,154 + 50,000 114,154

xxxviii. Answer: AThe amount of annual cash flows can be solved by equation:NPV = PV of annual CF – Investment1,750 = 2.4771CF – 2.4018CF1,750 = 0.1753CFCF = 9,980

xxxix . Answer: AInvestment 120,000Less Present value of salvage value (12,000 x 0.3855) 4,626Present value of Annual Cash Inflows 115,374Minimum Annual Cash Flows (115,374 ÷ 6.1446) 18,776

xl . Answer: BPresent value of annual cash flows at IRR (81,000 x 4.639) 375,759Investment 81,000 x 4.344 351,864Difference 23,895Annual increase in cash flows 23,895/4.344 5,501

xli . Answer: AInvestment (Total of present value @ IRR of 12%) 50,000Less PV, year 1 & 2 (16,074 + 17,534) 33,608PV of the 3rd cash flow 16,392After-tax cash flow, third year 16,392/0.712 23,022

xlii . Answer: BThe net present value = PV of excess salvage value less PV of decrease in after-tax cash flow

Let X = the excess salvage value7,003 = 0.56743X – [3.60478 x (0.2X * 0.4)7,003 = 0.56743X – 0.2883824X7,003 = 0.2790476X X = 25,096

Required salvage value: 50,000 – 25,096 = 24,904

Page 33: MODULE 9 CAPITAL BUDGETING THEORIES: Basic Concepts

xliii . Answer: BCost of equipment 750,000Less PV of tangible benefits 100,000 x 5.01877 501,877PV of annual intangible benefits 248,123Amount of annual intangible benefits 248,123/5.01877 49,440

xliv . Answer: BTo be acceptable, the project should yield a net present value of zero. The negative net present value must be offset bythe present value of annual intangible benefits.

Present value of intangible benefits P184,350PV of annuity of 1 at 10% for 10 years ÷ 6.145Annual net intangible benefits P30,000

xlv . Answer: AThe indifference rate (crossover or fisher rate) refers to the rate at which the net present values of the 2 alternatives are indifferent or equal.The easier test of the rate is to look for IRR (using trial and error technique) of the investment difference.

Difference 80,000 – 48,000 35,225PV inflows ∑(3,200 ÷ 1.1264)6 (12,922)PV inflows ∑(15,200 ÷ 1.1264)10-6 (22,303) Difference NIL

Alternative Solution:Project XProject YPV of after-tax cash flows

∑(12,000 ÷ 1.1264)648,455 ∑(15,200 ÷ 1.1264)1083,680Investment48,00083,225Net Present Value 455 455

xlvi . Answer: BThe determination of the indifference point, which is 10%, for the two projects can be made through the use of trial and error estimation. Machine 1Machine 2PV of Difference in ATCF Year 1 155,000 ÷ 1.10 140,909.10(140,909.10) Year 2 (110,000 ÷

1.10)2( 90,909.10) 90,909.10Net difference 50,000.00( 50,000.00)Difference in investment( 50,000.00)50,000.00NPV NIL NIL

xlvii . Answer: C15% Discount Rate

Machine 1Machine 2PV of Difference in ATCF Year 1 155,000 x 0.86957 134,783.35(134,783.35) Year 2 110,000 x0.75614( 83,175.40) 83,175.40Net difference 51,607.95( 51,607.95)Difference in investment( 50,000.00)50,000.00NPV 1,607.95( 1,607.95) At 15 percent discount rate, Machine 1 is more acceptable.

8% Discount RateMachine 1Machine 2PV of Difference in ATCF Year 1 155,000 x 0.92593 143,519.15(143,519.15) Year 2 110,000 x

0.85734( 94,307.40) 94,307.40Net difference 49,211.75( 49,211.75)Difference in investment( 50,000.00)50,000.00NPV ( 788.25) 788.25

At 8 percent discount rate, Machine 2 is more acceptable.

xlviii . Answer: CCost of Investment:

Invoice price 950,000Installation cost 24,200Freight charge 800 Total investment 975,000

Annual Cash Flow:Number of procedures: (52 x 5) 260Contribution margin per procedures: (P800 – P10 – P40) P750Total annual cash flow: (260 x P750) P195,000Cash payback period: (975,000 ÷ 195,000) 5 years

xlix . Answer: BPresent value of cash flow (195,000 x 6.418) P1,251,510Present value of salvage value (55,000 x 0.42241) 23,233Total P1,274,743Capital investment 975,000Net present value P 299,743

l . Answer: AAverage investment: (975,000 + 55,000) ÷ 2 515,000Annual depreciation: (975,000 – 55,000) ÷ 10 92,000Annual net income: 195,000 – 92,000 103,000Average annual Rate of Return: P103,000 P515,000 20%

Page 34: MODULE 9 CAPITAL BUDGETING THEORIES: Basic Concepts

li . Answer: A Contribution margin: 300,000 x (75 – 50) 7,500,000Less Fixed costs 4,500,000Cash flow before tax 3,000,000Less: Depreciation (6,000,000 ÷ 4) 1,500,000Income before tax 1,500,000Less: Income tax (1,500,000 x 0.4) 600,000Net income 900,000Add back: Depreciation 1,500,000After-tax Cash Flow 2,400,000

lii . Answer: CPV of After-tax Cash Flows (2,400,000 x 2.9287) 7,028,900Cost of investment 6,000,000Net Present Value 1,028,900

liii . Answer: AAnnual excess present value (1,028,000 ÷ 2.9287) P351,000Excess cash before tax (351,000 ÷ 0.6) P585,000Maximum number of units as decrease (585,000 ÷ 15) 39,000

liv . Answer: AAverage Annual net income:

(100,000 + 40,000 + 20,000 + 10,000 + 10,000) ÷ 5 = 36,000Divide by average investment (400,000 ÷ 2) 200,000Accounting rate of return 18%

Accounting rate of return or unadjusted rate of return computes the profitability of the project in term of accrual profit.Net profit under accrual method considers depreciation, a substantial amount that understates the average profit. Thisunderstatement of amount that is used in the computation necessarily requires that preferably, average investmentshould be used, instead of the initial investment, in the determination of accounting rate of return.

lv . Answer: BCash FlowPV FactorPV of annual net cash flows:180,0000.909163,620120,0000.826 99,120100,0000.751

75,10090,0000.683 61,47090,0000.621 55,890Total455,200Amount of investment400,000Net Present Value 55,200lvi . Answer: C

Present Value Index (Profitability Index) Present Value of ATCF ÷ Net Investment (455,200 ÷ 400,000) = 1.14

The present value index computes net present value in terms of P1 investment. Therefore, the index of 1.14 means thenet present value per P1 of investment is P0.14. This concept makes the present value index better than the netpresent value technique because the index indicates which one is the most profitable on a per P1 investment.

lvii . Answer: D Cash InflowUnrecovered InvestmentPeriod 0 Outflows(400,000)Period 1180,000(220,000)Period 2120,000(100,000)Period

3100,000ZeroThe total outflows are fully recovered by the end of period 3.The analyst should be careful in computing the payback period when the project has uneven cash inflows. The commonerror in handling uneven cash flows is using the average cash flows instead of reducing the unrecovered outflows.

lviii . Answer: DPayback period: Investment ÷ Net Annual Cash InflowP200,000 ÷ P50,000 = 4 years

lix . Answer: DPresent value of Net Cash Inflow (71,000 X 4.355) 309,205Investment 300,000Net Present value 9.205

lx . Answer: BAverage Investment: (200,000 ÷ 2) = 100,000Accounting Rate of Return = Net Income ÷ Average Investment (10,000 ÷ 100,000) = 10 percent

lxi . Answer: BThe payback for PA is 4.225. This is closest to the present value of annuity of 1 discounted at 11 percent for 6 periods which is 4.231.

lxii . Answer: AAnnual depreciation: (P50,000 ÷ 8) P6,250Annual tax shield: (P6,250 x 0.3) P1,875

Page 35: MODULE 9 CAPITAL BUDGETING THEORIES: Basic Concepts

lxiii . Answer: CBefore-tax cash inflow P10,000Less depreciation 6,250Income before tax 3,750Less income tax (3,750 x 0.3) 1,125Net income 2,625Add back depreciation 6,250After-tax cash inflow P 8,875

A quicker calculation of after tax cash flow can be made by adding the tax shield to after-tax cash inflow without any taxbenefit on depreciation.

(P10,000 × .70) + P1,875 = P8,875

lxiv . Answer: BPayback period: (P50,000 ÷ P8,875) = 5.6 years

lxv . Answer: CPresent value of annual ATCF (P8,875 x 5.747) P51,000Present value of after-tax salvage value (P1,400 x 0.54) 756 Total 51,756Investment 50,000Net present value P 1,756

lxvi . Answer: CAt the discount rate of 8 percent, there is a net present value of P1,756. Therefore, the IRR is higher than 8 percent. Using trial and error approach, the first try should use 9 percent. If the present value of the inflows exceeds P50,000,then the IRR is lower than 9 percent, otherwise it should be 9.5 percent.Using 9.0 percent in discounting the inflows, there is a net present value of P(174); therefore the IRR is slightly lowerthan but very close to 9.0 percent.(P8,875 x 5.535) + (P1,400 x 0.5019) – P50,000 = P(174)

lxvii . Answer: BAdditional contribution margin:Small 6,000 x 5.40 32,400Medium 15,000 x 6.50 97,500Large 9,000 x 7.90 71,100Total 201,000Less Cash Fixed Expenses:

Rent 18,000Salaries 54,000Utilities 13,200Insurance, etc. 7,800 93,000

Annual Cash Inflows 108,000

lxviii . Answer: BAdditional rental income 130,000Additional cash flow, snack bar 15,000Total 145,000Less Cash Fixed Expenses:

Rent 48,000Salaries 17,000Utilities 5,400Insurance, etc. 9,600 80,000

Annual Cash Inflow 65,000

lxix . Answer: APV of annual cash inflow (108,000 x 5.575) 602,100PV of salvage value (70,000 x 0.108) 3,240PV of working capital return (7,500 x 0.108) 810Total 606,150Investment:

Remodeling cost 550,000Working capital 7,500 557,500

Net Present Value 48,650

lxx . Answer: BPV of annual cash inflow (65,000 x 5.575) 362,375PV of salvage value 1,296PV of working capital return 432

Page 36: MODULE 9 CAPITAL BUDGETING THEORIES: Basic Concepts

Total 364,103Investment:

Remodeling cost 290,000Working capital 4,000 294,000

Net Present Value 70,103

lxxi . Answer: ARental income 21,000 x 5 105,000Additional cash inflow, snack bar 13,000Total 118,000Less fixed expenses 80,000Annual cash inflow 38,000

PV of annual cash inflow (38,000 x 5,575) 211,850PV of salvage value 1,296PV of working capital return 432Total 213,578Investment 294,000Negative Net Present Value ( 80,422)

lxxii . Answer: DThe annual cost of advertising can be easily calculated by dividing the net present value of alternative 2, at 16% by thepresent value of annuity of 1.

70,103 ÷ 5,575 = 12,574.53

lxxiii . Answer: AAnnual revenues 300,000Variable expenses 60,000Contribution margin 240,000Fixed expenses

Advertising 40,000Salaries 110,000Utilities 5,200Insurance 800 156,000

Annual cash income 84,000Less Depreciation 420,000 x 0.90 ÷ 12 31,500Annual Income 52,500

lxxiv . Answer: ACurrent operating costs – old machine 78,000Deduct Operating costs – Machine B

Annual salary of operator 16,350Annual maintenance cost 5,400 21,750

Annual cash savings 56,250

lxxv . Answer: ASimple Rate of Return = Net Income ÷ Initial Investment

52,500 ÷ 420,000 = 12.50 %

lxxvi . Answer: BSavings 56,250Less Depreciation 234000 ÷ 13 years 18,000Annual income 38,250Simple Annual Return 38,250 ÷ 225,000 17 %

lxxvii . Answer: CPayback period = Initial Investment ÷ Annual Cash Inflow

420,000 ÷ 84,000 = 5 years

lxxviii. Answer: A225,000 ÷ 56,250 = 4 years

lxxix . Answer: CPurchase price of new tools 2,500,000Add increase in working capital 60,000Total 2,560,000 Deduct Salvage value of the old tools 45,000Net investment 2,528,500

Page 37: MODULE 9 CAPITAL BUDGETING THEORIES: Basic Concepts

lxxx . Answer: CPurchase price of valve stem 80,000 x 20 1,600,000Cost to make:

Direct materials 80,000 x 4.50 360,000Direct labor 80,000 x 3.90 312,000Variable overhead 80,000 x 1.50 120,000

Decrease in directs labor and variable costs 80,000 x 1.60 (128,000) 664,000Cost savings 936,000

lxxxi . Answer: APV of annual depreciation

PeriodDepreciationPV FactorPresent ValueYear 1 832,5000.89286743,305.95 2 112,5000.79719886,873.88 3370,0000.71178263,358.60 4 185,0000.63552117,571.20Total2,011,109.63Tax rate0.30PV of tax benefits from

depreciation603,332.89lxxxii . Answer: C

After tax salvage value 100,000 x .7 70,000PV of 1 end of 4 periods 0.63552PV of after – tax salvage value 44,486.4

lxxxiii. Answer: CPV of after cash savings 936,000 x .7 x 3.03735 1990072PV of tax benefits from depreciation 603,333PV of after tax salvage value 44,486PV of working capital return 60,000 x 0.63552 38,131Investment (2528,500)Net present value 147,522

lxxxiv. Answer: APV of tax benefits, declining - balance 603,333PV of tax benefits, straight-line method 2,500,000 4 x .3 x 3.03735 569,503Net advantage 33,830

lxxxv . Answer: AInvoice price of new equipment (945,000 x 0.98) P926,100Freight 11,000Installation cost 22,900 Total 960,000 Less: Salvage value of old equipment (0.6 x 1,500) 900 Reduction in working capital 2,500 3,400Net initial outflows P956,600

lxxxvi. Answer: BTotal variable costs (262,000 units x P20*) P5,240,000Avoidable fixed costs (P45,000 x 5 years) 225,000Total 5,465,000

After-tax Cash outflowsOperating expenses (5,465,000 x 0.6) P3,279,000Depreciation (960,000 x 0.4) ( 384,000)After-tax salvage value of new equipment (12,000 x 0.60) ( 7,200) Net outflows P2,887,800

*Variable cost per unitDirect material (10.00 x 0.75) P 7.50Direct labor 8.00Variable overhead (6.00 x 0.75) 4.50 Total P20.00

lxxxvii. Answer: AThe present value of the tax shield based on declining-depreciation is:

YearDepreciationTax Shield (40%)PV FactorPV of Tax Shield2007P319,968P127,9870.893P114,2922008 426,720170,6880.797 136,0382009 142,176 56,8700.712 40,4922010 71,136 28,4550.636 18,098TotalP308,920

lxxxviii. Answer: CPurchase Cost

Year ATCF200750,000 x 27 x 0.6 810,000200850,000 x 27 x 0.6 810,000200952,000 x 27 x 0.6842,400201055,000 x 27 x 0.6 891,000201155,000 x 27 x 0.6 891,0002006(1,500 x 0.6) ( 900) Total

4,243,500lxxxix. Answer: A

Present value of after-tax cash flows

Page 38: MODULE 9 CAPITAL BUDGETING THEORIES: Basic Concepts

2007 (810,000 x 0.893) P 723,3302008 (810,000 x 0.797) 645,5702009 (842,400 x 0.712( 599,7892010 (891,000 x 0.636) 566,6762011 (891,000 x 0.567) 505,197Salvage value of old equipment (1,500 x 0.60) (900)Net present value P3,039,662

xc . Answer: DCFBTCFATPV FactorPVCFAT2006Initial outflow(P956,600)2007(50,000 x 20) + 45,000

(1,045,000 x 0.6) - (319,968 x 0.4)1,045,000499,013

0.893445,6192008(1,045,000 x 0.6) – (426,720 x 0.4)456,3120.797363,6812009(52,000 x 20) + 45,000

(1,085,000 x 0.6) – (142,176 x 0.4)1,085,000

594.1300.712

423,0212010(55,000 x 20) + 45,000(1,145,000 x 0.6) – (71,136)1,145,000

658,5460.636

418,8352011(55,000 x 20) + 45,000(1,145,000 x 0.6)1,145,000

687,0000.567

385,447Salvage value (12,000 x 0.6)7,200P2,993,203